LSAT and Law School Admissions Forum

Get expert LSAT preparation and law school admissions advice from PowerScore Test Preparation.

 Administrator
PowerScore Staff
  • PowerScore Staff
  • Posts: 8916
  • Joined: Feb 02, 2011
|
#38364
Complete Question Explanation
(The complete setup for this game can be found here: lsat/viewtopic.php?t=15015)

The correct answer choice is (A)

If S is seventh, then P must be sixth (third rule). Accordingly, the rotating PS block fully occupies the last two spaces on the diagram. The assignment of the remaining five variables will depend on the position of V, which is why we need to create two local diagrams, as shown below:
PT71_D13 LG Explanations_Game #4_#18_diagram 1.png
The solution to the first local setup is significantly simpler, because the R _ M block can only occupy positions 2-4. Thanks to the sequencing relationship between T and W, we can determine the positions of all seven variables:
PT71_D13 LG Explanations_Game #4_#18_diagram 2.png
The solution to the second setup is complicated by the fact that the R _ M block can be positioned in 1-3 or else 3-5. Once we take that into account, the remaining variables—T and W—must be sequenced in accordance with the first rule. Thus, the second local setup can be split into two different directions, as shown below:
PT71_D13 LG Explanations_Game #4_#18_diagram 3.png
These solutions reveal that T could be second, proving answer choice (A) correct.

The solution of this question would be much simpler with Templates: if S is seventh, only Templates 1, 2, and 3.2 represent a viable solution. Template 1 proves that T can be second.
You do not have the required permissions to view the files attached to this post.

Get the most out of your LSAT Prep Plus subscription.

Analyze and track your performance with our Testing and Analytics Package.